Easy , isn't it?

Algebra Level 3

{ a + b + c = 16 a 2 + b 2 + c 2 = 90 a 3 + b 3 + c 3 = 532 \large \begin{cases} a+b+c=16 \\ a^2+b^2+c^2=90 \\ a^3+b^3+c^3=532 \end{cases}

If a a , b b and c c satisfy the system of equations above, find a 4 + b 4 + c 4 a^4+b^4+c^4 .

You may use a calculator at the end. So best of luck!


The answer is 3282.

This section requires Javascript.
You are seeing this because something didn't load right. We suggest you, (a) try refreshing the page, (b) enabling javascript if it is disabled on your browser and, finally, (c) loading the non-javascript version of this page . We're sorry about the hassle.

3 solutions

Use Newton's Sums.

Let a 3 = 1 a_3 = 1

S 1 + a 2 = 0 a 2 = 16 S_1 + a_2 = 0 \Rightarrow a_2 = -16

S 2 + a 2 S 1 + 2 a 1 = 0 a 1 = 83 S_2 + a_2S_1 + 2a_1 = 0 \Rightarrow a_1 = 83

S 3 + a 2 S 2 + a 1 S 1 + 3 a 0 = 0 a 0 = 140 S_3 + a_2S_2 + a_1S_1 + 3a_0 = 0 \Rightarrow a_0 = -140

S 4 + a 2 S 3 + a 1 S 2 + a 0 S 1 = 0 S 4 = 3282 S_4 + a_2S_3 + a_1S_2 + a_0S_1 = 0 \Rightarrow S_4 = \boxed{3282}


Note:

  • S n = a n + b n + c n S_n = a^n + b^n + c^n
Majed Khalaf
Aug 20, 2015

( a + b + c ) 2 (a+b+c)^2 = ( a 2 + b 2 + c 2 ) + 2 × ( a b + b c + c a ) (a^2+b^2+c^2)+2 \times (ab+bc+ca) .

Let us substitute a + b + c = 16 a+b+c=16 and a 2 + b 2 + c 2 = 90 a^2+b^2+c^2=90 in the following equation and after that we get:

a b + b c + c a = 83... ( ) ab+bc+ca=83... (*)

( a + b + c ) × ( a 2 + b 2 + c 2 ) = ( a 3 + b 3 + c 3 ) + ( c a 2 + c b 2 + c 2 a + c 2 b + a 2 b + b 2 a ) (a+b+c)\times (a^2+b^2+c^2)=(a^3+b^3+c^3)+(ca^2+cb^2+c^2a+c^2b+a^2b+b^2a)

After substituting the values we get: c a 2 + c b 2 + c 2 a + c 2 b + a 2 b + b 2 a = 908... ( ) ca^2+cb^2+c^2a+c^2b+a^2b+b^2a=908...(**)

( a + b + c ) × ( a 2 + b 2 + c 2 ) = ( a 3 + b 3 + c 3 ) + 3 × ( c a 2 + c b 2 + c 2 a + c 2 b + a 2 b + b 2 a ) + 6 a b c (a+b+c)\times (a^2+b^2+c^2)=(a^3+b^3+c^3)+3\times (ca^2+cb^2+c^2a+c^2b+a^2b+b^2a)+6abc .

After substituting the values including ** we get: a b c = 140... ( ) abc=140...(***)

( a + b + c ) × ( a 3 + b 3 + c 3 ) = ( a 4 + b 4 + c 4 ) + a b × ( a 2 + b 2 ) + c a × ( a 2 + c 2 ) + b c × ( b 2 + c 2 ) = (a+b+c)\times (a^3+b^3+c^3)=(a^4+b^4+c^4)+ab\times (a^2+b^2)+ca\times (a^2+c^2)+bc\times (b^2+c^2)= ( a 4 + b 4 + c 4 ) + a b × ( 90 c 2 ) + c a × ( 90 b 2 ) + b c × ( 90 a 2 ) (a^4+b^4+c^4)+ab\times (90-c^2)+ca\times (90-b^2)+bc\times (90-a^2) = ( a 4 + b 4 + c 4 ) + 90 × ( a b + b c + c a ) a b c × ( a + b + c ) =(a^4+b^4+c^4)+90\times (ab+bc+ca)-abc\times (a+b+c)

Let us substitute * and * * and the values given in the question in the equation above and we're done: a 4 + b 4 + c 4 = 3282 a^4+b^4+c^4=3282 .

Chew-Seong Cheong
Jul 14, 2016

We can solve the problem using Newton's sums method. Let P n = a n + b n + c n P_n = a^n+b^n+c^n , S 1 = a + b + c = 16 S_1 = a+b+c = 16 , S 2 = a b + b c + c a S_2 = ab+bc+ca and S 3 = a b c S_3 = abc . Then, we have:

P 2 = P 1 S 1 2 S 2 a 2 + b 2 + c 2 = ( a + b + c ) ( a + b + c ) 2 S 2 90 = 1 6 2 2 S 2 S 2 = 83 P 3 = P 2 S 1 P 1 S 2 + 3 S 3 a 3 + b 3 + c 3 = ( 90 ) ( 16 ) ( 16 ) ( 83 ) + 3 S 3 532 = 112 + 3 S 3 S 3 = 140 P 4 = P 3 S 1 P 2 S 2 + P 1 S 3 a 4 + b 4 + c 4 = ( 532 ) ( 16 ) ( 90 ) ( 83 ) + ( 16 ) ( 140 ) = 3282 \begin{aligned} P_2 & = P_1S_1 - 2S2 \\ a^2 + b^2 + c^2 & = (a+b+c)(a+b+c) - 2S_2 \\ 90 & = 16^2 - 2S_2 \\ \implies S_2 & = 83 \\ P3 & = P_2S_1 - P_1S_2 + 3S_3 \\ a^3 + b^3 + c^3 & = (90)(16) - (16)(83) + 3S_3 \\ 532 & = 112 + 3S_3 \\ \implies S_3 & = 140 \\ \implies P_4 & = P_3S_1 - P_2S_2 + P_1S_3 \\ a^4 + b^4 + c^4 & = (532)(16) - (90)(83) + (16)(140) \\ & = \boxed{3282} \end{aligned}

0 pending reports

×

Problem Loading...

Note Loading...

Set Loading...